WBBSE Madhyamik Class 10(Ten)(X) Chapter 5 (Ratio Proportion) Exercise 5.3 Solution | অনুপাত সমানুপাত কষে দেখি ৫.৩ দশম শ্রেণী (ক্লাস ১০) | Koshe Dekhi 5.3 Class 10 | গণিত প্রকাশ দশম শ্রেণি অনুপাত সমানুপাত কষে দেখি 5.3 সমাধান|

Share this page using :

Koshe Dekhi 5.3 Class 10 | অনুপাত সমানুপাত কষে দেখি ৫.৩
কষে দেখি - 5.3

Koshe Dekhi 5.3 Class 10 | অনুপাত সমানুপাত কষে দেখি ৫.৩ | Class 10 Maths Solution WBBSE
Madhyamik Previous Year Question Paper | মাধ্যমিক বিগত বছরের অঙ্ক প্রশ্নের উত্তরপত্র
আজই Install করুন Chatra Mitra

1. \(a : b = c : d\) হলে, দেখাই যে,

(i) \((a^2 + b^2) : (a^2 - b^2) = (ac + bd) : (ac - bd)\)
মনে করা যাক, \(\frac{a}{b} = \frac{c}{d} = k(k \ne 0)\)
\(\therefore a=b k, c=d k\) (এখানে k একটি অশূন্য ধ্রুবক)
এখন, \(\frac{{{a^2} + {b^2}}}{{{a^2} - {b^2}}} = \frac{{{{(bk)}^2} + {b^2}}}{{{{(bk)}^2} - {b^2}}} = \frac{{{b^2}\left( {{k^2} + 1} \right)}}{{{b^2}\left( {{k^2} - 1} \right)}} = \frac{{{k^2} + 1}}{{{k^2} - 1}}(b \ne 0)\)
আবার, \(\frac{{ac + bd}}{{ac - bd}} = \frac{{bk \times dk + bd}}{{bk\times dk-bd}} = \frac{{bd\left( {{k^2} + 1} \right)}}{{bd\left( {{k^2} - 1} \right)}} = \frac{{{k^2} + 1}}{{{k^2} - 1}}{\rm{ }}(bd \ne 0)\)
সুতরাং হল যে, \(\left( {{a^2} + {b^2}} \right):\left( {{a^2} - {b^2}} \right) = (ac + bd):(ac - bd)\) (প্রমাণিত)
(ii) \(\left( {{a^2} + ab + {b^2}} \right):\left( {{a^2} - ab + {b^2}} \right) = \left( {{c^2} + cd + {d^2}} \right):\left( {{c^2} - cd + {d^2}} \right)\)
\(\frac{{{a^2} + ab + {b^2}}}{{{a^2} - ab + {b^2}}} = \frac{{{{(bk)}^2} + bk \times b + {b^2}}}{{{{(bk)}^2} - bk \times b + {b^2}}} = \frac{{{b^2}\left( {{k^2} + k + 1} \right)}}{{{b^2}\left( {{k^2} - k + 1} \right)}} = \frac{{{k^2} + k + 1}}{{{k^2} - k + 1}}\)
আবার, \(\frac{{{c^2} + cd + {d^2}}}{{{c^2} - cd + {d^2}}} = \frac{{{{(dk)}^2} + dk \times d + {d^2}}}{{{{(dk)}^2} - dk \times d + {d^2}}} = \frac{{{d^2}\left( {{k^2} + k + 1} \right)}}{{{d^2}\left( {{k^2} - k + 1} \right)}} = \frac{{{k^2} + k + 1}}{{{k^2} - k + 1}}\)
তাহলে প্রমাণিত হল যে, \(\left( {{a^2} + ab + {b^2}} \right):\left( {{a^2} - ab + {b^2}} \right) = \left( {{c^2} + cd + {d^2}} \right):\left( {{c^2} - cd + {d^2}} \right)\) (প্রমাণিত)
(iii) \(\sqrt {{a^2} + {c^2}} :\sqrt {{b^2} + {d^2}} = (pa + qc):(pb + qd)\)
\(\frac{{\sqrt {{a^2} + {c^2}} }}{{\sqrt {{b^2} + {d^2}} }} = \frac{{\sqrt {{(bk)}^2 + {{(dk)}^2}}}}{{\sqrt {{b^2} + {d^2}} }} = \frac{{\sqrt {{b^2}{k^2} + {d^2}{k^2}} }}{{\sqrt {{b^2} + {d^2}} }} = \frac{{k\sqrt {{b^2} + {d^2}} }}{{\sqrt {{b^2} + {d^2}} }} = k\)
আবার, \(\frac{{pa + qc}}{{pb + qd}} = \frac{{p.bk + qdk}}{{pb + qd}} = \frac{{k(pb + qd)}}{{pb + qd}} = k\)
তাহলে প্রমাণিত হল যে, \(\sqrt {{a^2} + {c^2}} :\sqrt {{b^2} + {d^2}} = (pa + qc):(pb + qd)\) (প্রমাণিত)

2. \(x : a = y : b = z : c\) হলে, প্রমাণ করি যে,

(i) \(\frac{{{x^3}}}{{{a^2}}} + \frac{{{y^3}}}{{{b^2}}} + \frac{{{z^3}}}{{{c^2}}} = \frac{{{{(x + y + z)}^3}}}{{{{(a + b + c)}^2}}}\)
\(x: a=y: b=z: c\)
মনে করা যাক, \(\frac{x}{a} = \frac{y}{b} = \frac{z}{c} = k(k \ne 0)\)
তাহলে \(x = ak,y = bk,z = ck\)
বামপক্ষ \( = \frac{{{x^3}}}{{{a^2}}} + \frac{{{y^3}}}{{{b^2}}} + \frac{{{z^3}}}{{{c^2}}} = \frac{{{{(ak)}^3}}}{{{a^2}}} + \frac{{{{(bk)}^3}}}{{{b^2}}} + \frac{{{{(ck)}^3}}}{{{c^2}}} = \frac{{{a^3}{k^3}}}{{{a^2}}} + \frac{{{b^3}{k^3}}}{{{b^2}}} + \frac{{{c^3}{k^3}}}{{{c^2}}}\)
\( = {a{k^3}} + b{k^3} + c{k^3} = {k^3}(a + b + c)\)
ডানপক্ষ \( = \frac{{{{(x + y + z)}^3}}}{{{{(a + b + c)}^2}}} = \frac{{{{(ak + bk + ck)}^3}}}{{{{(a + b + c)}^2}}} = \frac{{{k^3}{{(a + b + c)}^3}}}{{{{(a + b + c)}^2}}} = {k^3}(a + b + c)\)
সুতরাং, বামপক্ষ = ডানপক্ষ (প্রমাণিত)।
(ii) \(\frac{{{x^3} + {y^3} + {z^3}}}{{{a^3} + {b^3} + {c^3}}} = \frac{{xyz}}{{abc}}\)
বামপক্ষ \(\frac{{{x^3} + {y^3} + {z^3}}}{{{a^3} + {b^3} + {c^3}}} = \frac{{{{(ak)}^3} + {{(bk)}^3}{{(ck)}^3}}}{{{a^3} + {b^3} + {c^3}}} = \frac{{{k^3}\left( {{a^3} + {b^3} + {c^3}} \right)}}{{{a^3} + {b^3} + {c^3}}} = {k^3}\)
ডানপক্ষ \(\frac{{xyz}}{{abc}} = \frac{{ak \cdot bkck}}{{abc}} = \frac{{{k^3}abc}}{{abc}} = {k^3}\)
সুতরাং বামপক্ষ = ডানপক্ষ (প্রমাণিত)।
(iii) \(\left( {{a^2} + {b^2} + {c^2}} \right)\left( {{x^2} + {y^2} + {z^2}} \right) = {(ax + by + cz)^2}\)
বামপক্ষ \( = \left( {{a^2} + {b^2} + {c^2}} \right)\left( {{x^2} + {y^2} + {z^2}} \right)\)
\(=\left(a^2+b^2+c^2\right)\left\{(a k)^2+(b k)^2+(c k)^2\right\}\)
\( = \left( {{a^2} + {b^2} + {c^2}} \right)\left( {{a^2}{k^2} + {b^2}{k^2} + {c^2}{k^2}} \right)\)
\( = \left( {{a^2} + {b^2} + {c^2}} \right) \times {k^2} \times \left( {{a^2} + {b^2} + {c^2}} \right)\)
\( = {k^2}{\left( {{a^2} + {b^2} + {c^2}} \right)^2}\)
ডানপক্ষ \( = {(ax + by + cz)^2} = {(a \times ak + b \times bk + c \times ck)^2}\)
\( = {\left\{ {k\left( {{a^2} + {b^2} + {c^2}} \right)} \right\}^2} = {k^2}{\left( {{a^2} + {b^2} + {c^2} } \right)^2}\)
সুতরাং বামপক্ষ = ডানপক্ষ (প্রমাণিত)।

3. a : b = c : d = e : f হলে, প্রমাণ করি যে,

(i) প্রত্যেকটি অনুপাত \( = \frac{{5a - 7c - 13e}}{{5b - 7d - 13f}}\)
\(a: b=c: d=e: f\) [এখানে k একটি অশূন্য ধ্রুবক]
মনে করি,
\(\frac{a}{b}=\frac{c}{d}=\frac{e}{f}=k \)
\(\therefore a=b k, c=d k, e=f k\)
এখন, \(\frac{5 a-7 c-13 e}{5 b-7 d-13 f}=\frac{5 b k-7 d k-13 f k}{5 b-7 d-13 f}\)
\(=\frac{k(5 b-7 d-13 f)}{5 b-7 d-13 f}=k=\) প্রত্যেকটি অনুপাত।(প্রমাণিত)
Koshe Dekhi 5.3 Class 10 | অনুপাত সমানুপাত কষে দেখি ৫.৩ | Class 10 Maths Solution WBBSE
Madhyamik Previous Year Question Paper | মাধ্যমিক বিগত বছরের অঙ্ক প্রশ্নের উত্তরপত্র
আজই Install করুন Chatra Mitra
(ii) \(\left( {{{\rm{a}}^2} + {{\rm{c}}^2} + {{\rm{e}}^2}} \right)\left( {{{\rm{b}}^2} + {{\rm{d}}^2} + {{\rm{f}}^2}} \right) = {({\rm{ab}} + {\rm{cd}} + {\rm{ef}})^2}\)
বামপক্ষ\( = \left( {{a^2} + {c^2} + {e^2}} \right)\left( {{b^2} + {d^2} + {f^2}} \right)\)
\( = \left\{ {{{(bk)}^2} + {{(dk)}^2} + {{(fk)}^2}} \right\}\left( {{b^2} + {d^2} + {f^2}} \right)\)
\( = \left( {{b^2}{k^2} + {d^2}{k^2} + {f^2}{k^2}} \right)\left( {{b^2} + {d^2} + {f^2}} \right)\) \([\because a=b k, c=d k, e=f k]\)
\( = {{\rm{k}}^2}\left( {{b^2} + {d^2} + {f^2}} \right)\left( {{b^2} + {d^2} + {f^2}} \right) \)
\(= {k^2}{\left( {{b^2} + {d^2} + {f^2}} \right)^2}\)
ডানপক্ষ\( = {(ab + cd + {\rm{ ef }})^2}\)\( = {(bk \times b + dk \times d + fk \times f)^2}\)
\( = {\left\{ {k\left( {{b^2} + {d^2} + {f^2}} \right)} \right\}^2} = {k^2}{\left( {{b^2} + {d^2} + {f^2}} \right)^2}\)
\(\therefore\) বামপক্ষ = ডানপক্ষ (প্রমাণিত) ।

4. যদি \(a : b = b : c\) হয়, তবে প্রমাণ করি যে,

(i) \({\left( {\frac{{a + b}}{{b + c}}} \right)^2} = \frac{{{a^2} + {b^2}}}{{{b^2} + {c^2}}}\)
মনে করা যাক, \(\frac{a}{b} = \frac{b}{c} = k(k \ne 0)\)
\(\therefore b = ck\)এবং \(a = bk = ck.k = c{k^2}\)
বামপক্ষ \({\left( {\frac{{a + b}}{{b + c}}} \right)^2} = {\left( {\frac{{c{k^2} + ck}}{{ck + c}}} \right)^2} = {\left\{ {\frac{{ck(k + 1)}}{{c(k + 1)}}} \right\}^2} = \frac{{{c^2}{k^2}}}{{{c^2}}} = {k^2}\)
ডানপক্ষ \(\frac{{{a^2} + {b^2}}}{{{b^2} + {c^2}}} = \frac{{{{\left( {c{k^2}} \right)}^2} + {{(ck)}^2}}}{{{{(ck)}^2} + {c^2}}} = \frac{{{c^2}{k^4} + {c^2}{k^2}}}{{{c^2}{k^2} + {c^2}}} = \frac{{{c^2}{k^2}\left( {{k^2} + 1} \right)}}{{{c^2}\left( {{k^2} + 1} \right)}} = {k^2}\)
সুতরাং, বামপক্ষ = ডানপক্ষ (প্রমাণিত)।
(ii) \({a^2}{b^2}{c^2}\left( {\frac{1}{{{a^3}}} + \frac{1}{{{b^3}}} + \frac{1}{{{c^3}}}} \right) = {a^3} + {b^3} + {c^3}\)
যেহেতু a, b, c ক্রমিক সমানুপাতী, অতএব
\(\frac{a}{b}=\frac{b}{c}=k(\neq 0)\) (ধরি)
\(\therefore a=b k, \quad b=c k\)
অর্থাৎ \(a=c k . k=c k^{2}\) এবং \(b = ck\)
\(\therefore\) বামপক্ষ \(=a^{2} b^{2} c^{2}\left(\frac{1}{a^{3}}+\frac{1}{b^{3}}+\frac{1}{c^{3}}\right)\) \(=a^2 b^2 c^2\left(\frac{1}{\left(c k^2\right)^3}+\frac{1}{(c k)^3}+\frac{1}{c^3}\right)\)
\(=c^{2} k^{4} \times c^{2} k^{2} \times c^{2}\left(\frac{1}{c^{3} k^{6}}+\frac{1}{c^{3} k^{3}}+\frac{1}{c^{3}}\right)\)
\(=c^{6} k^{6} \times \frac{1}{c^{3}}\left(\frac{1}{k^{6}}+\frac{1}{k^{3}}+1\right)\)
\(=c^{3} k^{6} \times \frac{1+k^{3}+k^{6}}{k^{6}}\)
\(=c^{3}\left(1+k^{3}+k^{6}\right)\)
\(=c^{3}+c^{3} k^{3}+c^{3} k^{6}\)
\(=\left(c k^{2}\right)^{3}+(c k)^{3}+c^{3}\)
\(=a^{3}+b^{3}+c^{3}\)
\(\left(\because a=c k^{2}, b=c k\right)\)
\(\therefore\) বামপক্ষ = ডানপক্ষ (প্রমাণিত)
(iii) \(\frac{{abc{{(a + b + c)}^3}}}{{{{(ab + bc + ca)}^3}}} = 1\)
বামপক্ষ\( = \frac{{abc{{(a + b + c)}^3}}}{{{{(ab + bc + ca)}^3}}} \)
\(\left[\begin{array}{rl}\because a & =c k^2 \\ b & =c k\end{array}\right]\)
\(= \frac{{c{k^2} \times ck \times c{{\left( {c{k^2} + ck + c} \right)}^3}}}{{{{\left( {c{k^2} \times ck + ck \times c + c \times c{k^2}} \right)}^3}}}\)
\( = \frac{{{c^3}{k^3}{{\left\{ {c\left( {{k^2} + k + 1} \right)} \right\}}^3}}}{{{{\left( {{c^2}{k^3} + {c^2}k + {c^2}{k^2}} \right)}^3}}} = \frac{{{c^6}{k^3}{{\left( {{k^2} + k + 1} \right)}^3}}}{{{{\left\{ {{c^2}k\left( {{k^2} + 1 + k} \right)} \right\}}^3}}}\)
\( = \frac{{{c^6}{k^3}{{\left( {{k^2} + k + 1} \right)}^3}}}{{{c^6}{k^3}{{\left( {{k^2} + k + 1} \right)}^3}}} = 1\)(প্রমাণিত)।

5. \(a, b, c, d\) ক্ৰমিক সমানুপাতী হলে, প্রমাণ করি যে,

(i) \(\left( {{a^2} + {b^2} + {c^2}} \right)\left( {{b^2} + {c^2} + {d^2}} \right) = {(ab + bc + cd)^2}\)
\(a, b, c, d\)
যেহেতু, a, b, c, d ক্ৰমিক সমানুপাতী,
\(\frac{a}{b} = \frac{b}{c} = \frac{c}{d} = k\)মনে করা যাক
\(\therefore c = dk, b = ck = dk \times k =dk^2,a = bk = dk^2 \times k ={k}^3\)
বামপক্ষ\( = \left( {{a^2} + {b^2} + {c^2}} \right)\left( {{b^2} + {c^2} + {d^2}} \right)\)
\( = \left\{ {{{\left( {d{k^3}} \right)}^2} + {{\left( {d{k^2}} \right)}^2} + {{(dk)}^2}} \right\}\left\{ {{{\left( {d{k^2}} \right)}^2} + {{(dk)}^2} + {d^2}} \right\}\)
\( = \left\{ {{{\rm{d}}^2}{{\rm{k}}^6} + {{\rm{d}}^2}{{\rm{k}}^4} + {{\rm{d}}^2}{{\rm{k}}^2}} \right\}\left\{ {{{\rm{d}}^2}{{\rm{k}}^4} + {{\rm{d}}^2}{{\rm{k}}^2} + {{\rm{d}}^2}} \right\}\)
\( = {{\rm{d}}^2}{{\rm{k}}^2}\left( {{{\rm{k}}^4} + {{\rm{k}}^2} + 1} \right).{{\rm{d}}^2}\left( {{{\rm{k}}^4} + {{\rm{k}}^2} + 1} \right)\)
\(= {{\rm{d}}^4}{{\rm{k}}^2}{\left( {{{\rm{k}}^4} + {{\rm{k}}^2} + 1} \right)^2}\)
ডানপক্ষ \( = {(ab + bc + cd)^2}\)
\(= {\left\{ {d{k^3} \times d{k^2} + d{k^2} \times dk + dk \times d} \right\}^2}\)
\( = {\left( {{d^2}{k^5} + {d^2}{k^3} + {d^2}k} \right)^2}\)
\(= {\left\{ {{d^2}k\left( {{k^4} + {k^2} + 1} \right)} \right\}^2}\)
\( = {d^4}{k^2}{\left( {{k^4} + {k^2} + 1} \right)^2}\)
\(\therefore\) বামপক্ষ = ডানপক্ষ (প্রমাণিত)।
Koshe Dekhi 5.3 Class 10 | অনুপাত সমানুপাত কষে দেখি ৫.৩ | Class 10 Maths Solution WBBSE
Madhyamik Previous Year Question Paper | মাধ্যমিক বিগত বছরের অঙ্ক প্রশ্নের উত্তরপত্র
আজই Install করুন Chatra Mitra
(ii) \({(b - c)^2} + {(c - a)^2} + {(b - d)^2} = {(a - d)^2}\)
বামপক্ষ \( = {(b - c)^2} + {(c - a)^2} + {(b - d)^2}\)
\( = {\left( {d{k^2} - dk} \right)^2} + {\left( {dk - d{k^3}} \right)^2} + {\left( {d{k^2} - d} \right)^2}\) \(\left[\because a=d k^3, b=d k^2, c=d k\right]\)
\( = {\{ dk(k - 1)\} ^2} + \{ dk{\left( {1 - {k^2}} \right)\}^2} + {\left\{ {d\left( {{k^2} - 1} \right)} \right\}^2}\)
\( = {{\rm{d}}^2}{k^2}\left( {{k^2} - 2k + 1} \right) + {d^2}{k^2}\left( {1 - 2{k^2} + {k^4}} \right) + {d^2}\left( {{k^4} - 2{k^2} + 1} \right)\)
\( = {{\rm{d}}^2}{k^2}\left( {{k^2} - 2k + 1} \right) + {d^2}{k^2}\left( {1 - 2{k^2} + {k^4}} \right) + {d^2}\left( {{k^4} - 2{k^2} + 1} \right)\)
\( = {{\rm{d}}^2}\left\{ {{{\rm{k}}^4} - \left( {2{{\rm{k}}^3}} \right) + {{\rm{k}}^2} + {{\rm{k}}^2} - 2{{\rm{k}}^4} + \left( {{{\rm{k}}^6}} \right) + {{\rm{k}}^4} - 2{{\rm{k}}^2} + 1} \right\}\)
\( = {d^2}\left( {{k^6} - 2{k^3} + 1} \right)\)
\(=d^2\left\{\left(k^3\right)^2-2 \times k^3 \times 1+(1)^2\right\}\)
\(= {d^2}{\left( {{k^3} - 1} \right)^2}\)
ডানপক্ষ\( = {(a - d)^2}\)
\(= {\left( {d{k^3} - d} \right)^2}\)
\(= {\left\{ {d\left( {{k^3} - 1} \right)} \right\}^2}\)
\(= {d^2}{\left( {{k^3} - 1} \right)^2}\)
\(\therefore\) বামপক্ষ = ডানপক্ষ (প্রমাণিত)।

6.

(i) যদি \(\frac{m}{a} = \frac{n}{b}\)হয়, তবে দেখাই যে, \(\left( {{m^2} + {n^2}} \right)\left( {{a^2} + {b^2}} \right) = {(am + bn)^2}\)
\(\frac{m}{a} = \frac{n}{b} = k(k \ne 0)\) ধরা যাক তাহলে, \({\rm{m}} = {\rm{ak}},{\rm{n}} = {\rm{bk}}\)
বামপক্ষ\( = \left( {{m^2} + {n^2}} \right)\left( {{a^2} + {b^2}} \right)\)
\(= \left\{ {{{(ak)}^2} + {{(bk)}^2}} \right\}\left( {{a^2} + {b^2}} \right)\) \([\because m=a k, n=b k]\)
\( = \left( {{a^2}{k^2} + {b^2}{k^2}} \right)\left( {{a^2} + {b^2}} \right)\)
\( = {k^2}\left( {{a^2} + {b^2}} \right)\left( {{a^2} + {b^2}} \right)\)
\( = {k^2}{\left( {{a^2} + {b^2}} \right)^2}\)
ডানপক্ষ\( = {(am + bn)^2}\)
\( = {(a \times ak + b \times bk)^2}\)
\( = {\left\{ {k\left( {{a^2} + {b^2}} \right)} \right\}^2}\)
\( = {{\rm{k}}^2}{\left( {{{\rm{a}}^2} + {{\rm{b}}^2}} \right)^2}\)
\(\therefore\) বামপক্ষ = ডানপক্ষ (প্রমাণিত)
অর্থাৎ, \(\left( {{m^2} + {n^2}} \right)\left( {{a^2} + {b^2}} \right) = {(am + bn)^2}\) (প্রমাণিত)।
(ii) যদি \(\frac{a}{b} = \frac{x}{y}\) হয়, তবে দেখাই যে, \((a + b)\left( {{a^2} + {b^2}} \right){x^3} = (x + y)\left( {{x^2} + {y^2}} \right){a^3}\)
এখানে দেওয়া আছে \(\frac{a}{b} = \frac{x}{y}\)
বা, \(\frac{a}{x} = \frac{b}{y} = k( \ne 0)\) হলে ধরা যাক
\(\therefore a = kx,b = ky\)
বামপক্ষ \(=(a y+b)\left(a^2+b^2\right) x^3\)
\(=(k x+k y)\left\{k^2\left(x^2+y^2\right)\right\} \times x^3\)
\( = k(x + y)\left\{ {{k^2}\left( {{x^2} + {y^2}} \right)} \right\} \cdot {x^3}\)
\( = (a + b)\left( {{a^2} + {b^2}} \right){x^3}\)
\(= (kx + ky)\left\{ {{{(kx)}^2} + {{(ky)}^2}} \right\}{x^3}\)
\(=k(x+y) \{k^{2} \left( x^{2}+y^{2}\right)\} \cdot x^{3}\)
\( = {k^3}(x + y)\left( {{x^2} + {y^2}} \right){x^3}\)
\( = (x + y)\left( {{x^2} + {y^2}} \right){(kx)^3}\)
\( = (x + y)\left( {{x^2} + {y^2}} \right){(a)^3}\)
\(= (x + y)\left( {{x^2} + {y^2}} \right){a^3}\)
= ডানপক্ষ (প্রমাণিত)।
\(\therefore\) বামপক্ষ = ডানপক্ষ (প্রমাণিত)।
(iii) যদি \(\frac{x}{{\ell m - {n^2}}} = \frac{y}{{mn - {\ell ^2}}} = \frac{z}{{n\ell - {m^2}}}\) হয়, তবে দেখাই যে, \({\rm{ }}\ell x{\rm{ + m y + n z = 0}}\)
ধরি, \(\frac{x}{l m-n^{2}}=\frac{y}{m n-l^{2}}=\frac{z}{n l-m^{2}}=k\)
\(\therefore \quad x=k\left(l m-n^{2}\right), y=k\left(m n-l^{2}\right), z=k\left(n l-m^{2}\right)\)
এখন, \(l x+m y+n z\)
\(=l k\left(l m-n^{2}\right)+m k\left(m n-l^{2}\right)+n k\left(n l-m^{2}\right)\)
\(=k\left\{l\left(l m-n^{2}\right)+m\left(m n-l^{2}\right)+n\left(n l-m^{2}\right)\right\}\)
\(=k\left(l^{2} m-ln ^{2}+m^{2} n-l^{2} m+n^{2} l-m^{2} n\right)\)
\(=k \times 0=0\)
অতএব, \(l x+m y+n z=0\) (প্রমাণিত)
(iv) \(\frac{x}{{b + c - a}} = \frac{y}{{c + a - b}} = \frac{z}{{a + b - c}}\) হলে, দেখাই যে, \({\rm{( b - c ) x + ( c - a ) y + ( a - b ) z = 0}}\)
মনে করা যাক, \(\frac{x}{{b + c - a}} = \frac{y}{{c + a - b}} = \frac{z}{{a + b - c}} = k( \ne 0)\)
\(\therefore x = k(b + c - a),y = k(c + a - b),z = k(a + b - c)\)
এখন, \({\rm{(b - c ) }}x{\rm{ + ( c - a ) y + ( a - b ) z}}\)
\({\rm{ = ( b - c ) }}{\rm{ k ( b + c - a ) + ( c - a ) k ( c + a - b ) + ( a - b ) k ( a + b - c )}}\)
\( = k\{ (b - c)(b + c - a) + (c - a)(c + a - b) + (a - b)(a + b - c)\} \)
\( = k \{ (b - c)(b + c) - a(b - c) + (c - a)(c + a) - b(c - a) + (a - b)(a + b) - c(a - b)\}\)
\( = k\left\{ {{b^2} - {c^2} - ab + ca + {c^2} - {a^2} - bc + ab + {a^2} - {b^2} - ca + bc} \right\} = k \times 0\)
\(= 0\)(প্রমাণিত)
(v) \(\frac{x}{y} = \frac{{a + 2}}{{a - 2}}\) হলে, দেখাই যে, \(\frac{{{x^2} - {y^2}}}{{{x^2} + {y^2}}} = \frac{{4a}}{{{a^2} + 4}}\)
দেওয়া আছে, \(\frac{x}{y} = \frac{{a + 2}}{{a - 2}}\) দুদিকের বর্গ করলে, \(\frac{{{x^2}}}{{{y^2}}} = \frac{{{{(a + 2)}^2}}}{{{{(a - 2)}^2}}}\)
বা, \(\frac{{{x^2} + {y^2}}}{{{x^2} - {y^2}}} = \frac{{{{(a + 2)}^2} + {{(a - 2)}^2}}}{{{{(a + 2)}^2} - {{(a - 2)}^2}}}\) [উভয়দিকে যোগ-ভাগ করে পাই]
\( = \frac{{{a^2} + 4a + 4 + {a^2} - 4a + 4}}{{{a^2} + 4a + 4 - {a^2} + 4a - 4}}\)
\( = \frac{{2\left( {{a^2} + 4} \right)}}{{8a}}\)
\(= \frac{{{a^2} + 4}}{{4a}}\)
\(\therefore\) \(\frac{{{x^2} - {y^2}}}{{{x^2} + {y^2}}} = \frac{{4a}}{{{a^2} + 4}}\) (প্রমাণিত)
(vi) \(x = \frac{{8ab}}{{a + b}}\) হালে, \(\left( {\frac{{x + 4a}}{{x - 4a}} + \frac{{x + 4b}}{{x - 4b}}} \right)\)-এর মান হিসাব করে লিখি ।
\(x = \frac{{8ab}}{{a + b}}\)
বা, \(\frac{x}{{4a}} = \frac{{2b}}{{a + b}}\) (উভয় পক্ষকে 4a দ্বারা ভাগ করে পাই)
সুতরাং \(\frac{{x + 4a}}{{x - 4a}} = \frac{{2b + a + b}}{{2b - a - b}}\) [যোগ-ভাগ প্রক্রিয়ার সাহায্যে পাই]
\( = \frac{{3b + a}}{{b - a}}\)
আবার, \(x = \frac{{8ab}}{{a + b}}\)
বা, \(\frac{x}{{4b}} = \frac{{2a}}{{a + b}}\) (উভয়পক্ষকে 4b দ্বারা ভাগ করে পাই)
সুতরাং \(\frac{{x + 4b}}{{x - 4b}} = \frac{{2a + a + b}}{{2a - a - b}} = \frac{{3a + b}}{{a - b}}\) [জগ-ভাগ প্রক্রিয়ার সাহায্যে পাই]
\(\therefore\) \(\frac{{x + 4a}}{{x - 4a}} + \frac{{x + 4b}}{{x - 4b}}\) \(= \frac{{3b + a}}{{b - a}} + \frac{{3a + b}}{{a - b}}\)
\(=\frac{3 b+a}{b-a}+\frac{3 a+b}{-(b-a)}\)
\(= \frac{{3{b} + a}}{{b - a}} - \frac{{3a + b}}{{b - a}}\)
\( = \frac{{3b + a - 3a - b}}{{b - a}}\)
\(= \frac{{2b - 2a}}{{b - a}}\)
\(= \frac{{2(b - a)}}{{(b - a)}}\)
\(\therefore\) নির্ণেয় মান \(= 2\)
Koshe Dekhi 5.3 Class 10 | অনুপাত সমানুপাত কষে দেখি ৫.৩ | Class 10 Maths Solution WBBSE
Madhyamik Previous Year Question Paper | মাধ্যমিক বিগত বছরের অঙ্ক প্রশ্নের উত্তরপত্র
আজই Install করুন Chatra Mitra

7.

(i)\(\frac{a}{3} = \frac{b}{4} = \frac{c}{7}\) হলে, দেখাই যে, \(\frac{{a + b + c}}{c} = 2\)
মনে করি, \(\frac{a}{3}=\frac{b}{4}=\frac{c}{7}=k(\neq 0)\)
\(\therefore a=3 k, \quad b=4 k, \quad c=7 k\)
সুতরাং, \(\frac{a+b+c}{c}=\frac{3 k+4 k+7 k}{7 k}=\frac{14 k}{7 k}=2\)
\(\therefore\) নির্ণেয় মান \(= 2\) (প্রমাণিত)
(ii) \(\frac{a}{{q - r}} = \frac{b}{{r - p}} = \frac{c}{{p - q}}\) হলে,দেখাই যে, \({\rm{a + b + c = 0 = p a + q b + r c}}\)
মনে করা যাক, \(\frac{a}{q-r}=\frac{b}{r-p}=\frac{c}{p-q}=k\) \((\because k \neq 0)\)
তাহলে, \({\rm{a = k ( q - r ) , b = k ( r - p ) , c = k ( p - q )}}\)
এখন, \({\rm{a + b + c = k ( q - r ) + k ( r - p ) + k ( p - q )}}\)
\( = {\rm{k}}({\rm{q}} - {\rm{r}} + {\rm{r}} - {\rm{p}} + {\rm{p}} - {\rm{q}}) = {\rm{k}} \times 0 = 0\)
আবার, \({\rm{pa}} + {\rm{qb}} + {\rm{rc}} = {\rm{p}} \times {\rm{k}}({\rm{q}} - {\rm{r}}) + q \times {\rm{k}}({\rm{r}} - {\rm{p}}) + {\rm{r}} \times k({\rm{p}} - {\rm{q}})\)
\( = {\rm{k}}\{ {\rm{p}}({\rm{q}} - {\rm{r}}) + {\rm{q}}({\rm{r}} - {\rm{p}}) + {\rm{r}}({\rm{p}} - q)\}\)
\( = k(pq - rp + qr - pq + rp - qr) = k \times 0 = 0\)
তাহলে দেখানো হল যে, \({\rm{a + b + c = 0 = p a + q b + r c}}\) (প্রমাণিত)
(iii) \(\frac{{ax + by}}{a} = \frac{{bx - ay}}{b}\) হলে, দেখাই যে প্রতিটি অনুপাত \(x\)-এর সমান।
প্রদত্ত \(\frac{a x+b y}{a}=\frac{b x-a y}{b}\)
বা, \(\frac{a(a x+b y)}{a \cdot a}=\frac{b(b x-a y)}{b b}\)
বা, \(\frac{a^{2} x+a b y}{a^{2}}=\frac{b^{2} x-a b y}{b^{2}}=\frac{a^{2} x+a b y+b^{2} x-a b y}{a^{2}+b^{2}}\) [সংযােজন প্রক্রিয়া।]
\(=\frac{\left(a^{2}+b^{2}\right) x}{a^{2}+b^{2}}=x\)
\(\therefore \frac{a x+b y}{a}=\frac{b x-a y}{b}=x\)(প্রমাণিত)
বিকল্প পদ্ধতি :
মনে করি, \(\frac{a x+b y}{a}=\frac{b x-a y}{b}=k\)\([\because k \neq 0]\)
\(\therefore k a=a x+b y, k b=b x-a y\)
\(\therefore k a \cdot a+k b b=a(a x+b y)+b(b x-a y)\)
বা, \(k\left(a^{2}+b^{2}\right)=a^{2} x+a b y+b^{2} x-a b y\)
\(=a^{2} x+b^{2} x=x\left(a^{2}+b^{2}\right)\)
\(\therefore k=\frac{x\left(a^{2}+b^{2}\right)}{a^{2}+b^{2}}=x\)
\(\therefore\) প্রতিটি অনুপাত = \(k = x\) (প্রমাণিত)

8.

(i) যদি \(\frac{{a + b}}{{b + c}} = \frac{{c + d}}{{d + a}}\) হয়, তবে প্রমাণ করি যে, \(c = a\) অথবা \(a + b + c + d = 0\)
একান্তর প্রক্রিয়ানুসারে, \(\frac{a+b}{c+d}=\frac{b+c}{d+a}\)
বা, উভয়পক্ষে 1 যােগ করে পাই, \(\frac{a+b}{c+d}+1=\frac{b+c}{d+a}+1\)
বা, \(\frac{a+b+c+d}{c+d}=\frac{a+b+c+d}{d+a}\)
বা, \((a+b+c+d)\left(\frac{1}{c+d}-\frac{1}{d+a}\right)=0\)
\(\therefore\) হয় \(a + b + c + d = 0 \cdots(i)\)
বা, \(\frac{1}{c+d}=\frac{1}{d+a}\)
অর্থাৎ, \(c+ d = d + a \cdots(ii)\)
অর্থাৎ, c = a
অতএব, (i) এবং (ii) থেকে
হয় \(c = a\) কিংবা \(a = b + c+ d = 0\) (প্রমাণিত)
(ii) যদি \(\frac{x}{{b + c}} = \frac{y}{{c + a}} = \frac{z}{{a + b}}\) হয়, দেখাই যে, \(\frac{a}{{y + z - x}} = \frac{b}{{z + x - y}} = \frac{c}{{x + y - z}}\)
মনে করি, \(\frac{x}{b+c}=\frac{y}{c+a}=\frac{2}{a+b}=k\) [যেখানে k একটি সমানুপাতিক ধ্রুবক]
\(\therefore x=k(b+c), y=k(c+a), z=k(a+b)\)
এখন, \(\frac{a}{y+z-x}\)
\(=\frac{a}{k(c+a)+k(a+b)-k(b+c)}\)
\(=\frac{a}{k(c+a+a+b-b-c)}=\frac{a}{k \cdot 2 a}=\frac{1}{2 k}\)
আবার, \(\frac{b}{z+x-y}\) \(=\frac{b}{k(a+b)+k(b+c)-k(c+a)}\)
\(=\frac{b}{k(a+b+b+c-c-a)}=\frac{b}{k \cdot 2 b}=\frac{1}{2 k}\)
\(\frac{c}{x+y-z}\)
আবার, \(=\frac{c}{k(b+c)+k(c+a)-k(a+b)}\)
\(=\frac{c}{k(b+c+c+a-a-b)}=\frac{c}{k \cdot 2 c}=\frac{1}{2 k}\)
\(\therefore \frac{a}{y+z-x}=\frac{b}{z+x-y}=\frac{c}{x+y-z}=\frac{1}{2 k}\)
অতএব, \(\frac{a}{x+z-x}=\frac{b}{z+x-y}=\frac{c}{x+y-z}\) (প্রমাণিত)
(iii) \(\frac{{x + y}}{{3a - b}} = \frac{{y + z}}{{3b - c}} = \frac{{z + x}}{{3c - a}}\) হলে, দেখাই যে, \(\frac{{x + y + z}}{{a + b + c}} = \frac{{ax + by + cz}}{{{a^2} + {b^2} + {c^2}}}\)
মনে করি, \(\frac{x+y}{3 a-b}=\frac{y+z}{3 b-c}=\frac{z+x}{3 c-a}=k\) [যেখানে k একটি সমানুপাতিক ধ্রুবক]
\(\therefore x+y=k(3 a-b), y+z=k(3 b-c), z+x=k(3 c-a)\)
\(\therefore(x+y)+(y+z)+(z+x)=k\{(3 a-b)+(3 b-c)+(3 c-a)\}\)
বা, \(2(x+y+z) = 2k(a+b+c)\)
বা, \(x+y+z = k(a+b+c)\).....(1)
\(\therefore x=k(a+b+c)-k(3 b-c)=k(a-2 b+2 c)\)
\(y=k(a+b+c)-k(3 c-a)=k(2 a+b-2 c)\)
\(z=k(a+b+c)-k(3 a-b)=k(2 b-2 a+c)\)
\(\therefore \frac{a x+b y+c z}{a^{2}+b^{2}+c^{2}}\)
\(=\frac{a\{k(a-2 b+2 c)\}+b\{k(2 a+b-2 c)\}+c\{k(2 b-2 a+c)\}}{a^2+b^2+c^2}\)
\(=\frac{k\left(a^{2}-2 a b+2 a c+2 a b+b^{2}-2 b c+2 b c-2 a c+c^{2}\right)}{a^{2}+b^{2}+c^{2}}\)
\(=\frac{k\left(a^2+b^2+c^2\right)}{a^2+b^2+c^2}\)
\(=\frac{k\left(a^{2}+b^{2}+c^{2}\right)}{a^{2}+b^{2}+c^{2}}=k \ldots \ldots\) (2)
(1) ও (2) থেকে পাই \(\frac{x+y+z}{a+b+c}=k=\frac{a x+b y+c z}{a^{2}+b^{2}+c^{2}}\)
অর্থাৎ, \(\frac{x+y+z}{a+b+c}=\frac{a x+b y+c z}{a^{2}+b^{2}+c^{2}}\) (প্রমাণিত)\(\therefore\)
(iv) \(\frac{x}{a} = \frac{y}{b} = \frac{z}{c}\) হলে, তবে দেখাই যে, \(\frac{{{x^2} - yz}}{{{a^2} - bc}} = \frac{{{y^2} - zx}}{{{b^2} - ca}} = \frac{{{z^2} - xy}}{{{c^2} - ab}}\)
মনে করা যাক, \(\frac{x}{a} = \frac{y}{b} = \frac{z}{c} = k( \ne 0)\) [যেখানে k একটি সমানুপাতিক ধ্রুবক]
\(\therefore x=a k, y=b k, z=c k\)
এখন, \(\frac{{{x^2} - yz}}{{{a^2} - bc}} = \frac{{{{(ak)}^2} - bkck}}{{{a^2} - bc}} = \frac{{{k^2}\left( {{a^2} - bc} \right)}}{{{a^2} - bc}} = {k^2}\)
আবার, \(\frac{{{y^2} - zx}}{{{b^2} - ca}} = \frac{{{{(bk)}^2} - ckak}}{{{b^2} - ca}} = \frac{{{k^2}\left( {{b^2} - ca} \right)}}{{{b^2} - ca}} = {k^2}\)
এবং \(\frac{{{z^2} - xy}}{{{c^2} - ab}} = \frac{{{{(ck)}^2} - ak \cdot bk}}{{{c^2} - ab}} = \frac{{{k^2}\left( {{c^2} - ab} \right)}}{{{c^2} - ab}} = {k^2}\)
সুতরাং প্রমাণিত হল যে, \(\frac{{{x^2} - yz}}{{{a^2} - bc}} = \frac{{{y^2} - zx}}{{{b^2} - ca}} = \frac{{{z^2} - xy}}{{{c^2} - ab}}\)

9.

(i) যদি \(\frac{{3x + 4y}}{{3u + 4v}} = \frac{{3x - 4y}}{{3u - 4v}}\) হয়, তবে দেখাই যে, \(\frac{x}{y} = \frac{u}{v}\)
\(\frac{{3x + 4y}}{{3u + 4v}} = \frac{{3x - 4y}}{{3u - 4v}}\)
বা, \(\frac{{3x + 4y}}{{3x - 4y}} = \frac{{3u + 4v}}{{3u - 4v}}\)
বা, \(\frac{{3x + 4y + 3x - 4y}}{{3x + 4y - 3x + 4y}} = \frac{{3u + 4v + 3u - 4v}}{{3u + 4v - 3u + 4v}}\)(যোগ-ভাগ প্রক্রিয়া)
বা, \(\frac{{6x}}{{8y}} = \frac{{6u}}{{8v}}\)
বা, \(\frac{x}{y} = \frac{u}{v}\) [প্রমাণিত]
\(\therefore \frac{x}{y}=\frac{u}{v}\)
(ii) \(( a + b + c + d ) : ( a + b - c - d ) = ( a - b + c - d ) : ( a - b - c + d )\) হলে, প্রমাণ করি যে, \(a : b =c : d\)
\({\rm{( a + b + c + d ) : ( a + b - c - d ) = ( a - b + c - d ) : ( a - b - c + d )}}\)
বা, \(\frac{{a + b + c + d}}{{a + b - c - d}} = \frac{{a - b + c - d}}{{a - b - c + d}}\)
বা, \(\frac{{a + b + c + d + a + b - c - d}}{{a + b + c + d - a - b + c + d}} = \frac{{a - b + c - d + a - b - c + d}}{{a - b + c - d - a + b + c - d}}\) (যোগ-ভাগ প্রক্রিয়া)
বা, \(\frac{{2a + 2b}}{{2c + 2d}} = \frac{{2a - 2b}}{{2c - 2d}}\)
বা, \(\frac{2(a+b)}{2(c+d)}=\frac{2(a-b)}{2(c-d)}\)
বা, \(\frac{{a + b}}{{c + d}} = \frac{{a - b}}{{c - d}}\)
বা, \((a+b)(c-d)=(a-b)(c+d)\)
বা, \({\rm{a c - a d + b c - b d = a c + a d - b c - b d}}\)
\(a c+b c-b d-a c+b c+d b=a d+a d\)
বা, \({\rm{2 b c = 2 a d}}\)
বা, \({\rm{a d = b c}}\)
বা, \(\frac{a}{b} = \frac{c}{d}\)
\(\therefore a:b = c:d\)(প্রমাণিত)
Koshe Dekhi 5.3 Class 10 | অনুপাত সমানুপাত কষে দেখি ৫.৩ | Class 10 Maths Solution WBBSE
Madhyamik Previous Year Question Paper | মাধ্যমিক বিগত বছরের অঙ্ক প্রশ্নের উত্তরপত্র
আজই Install করুন Chatra Mitra

10.

(i) \(\frac{{{a^2}}}{{b + c}} = \frac{{{b^2}}}{{c + a}} = \frac{{{c^2}}}{{a + b}} = 1\) হলে, দেখাই যে \(\frac{1}{{1 + a}} + \frac{1}{{1 + b}} + \frac{1}{{1 + c}} = 1\)
\(\frac{a^2}{b+c}=\frac{b^2}{a+a}=\frac{c^2}{a+b}=1\)
\(\therefore\) \(a^{2}=b+c, b^{2}=c+a, c^{2}=a+b\)
এখন, \(a^{2}=b+c\)
\(\therefore\) \(a+a^{2}=a+b+c\) [উভয় পক্ষের সঙ্গে \(a\) যােগ করে]
বা, \(a(1+a)=a+b+c\)
\(\therefore \frac{1}{1+a}=\frac{a}{a+b+c}\) ............ (i)
আবার, \(b^{2}=c+a\), ইহার উভয় পক্ষের সঙ্গে \(b\) যােগ করে পাই,
\(b+b^{2}=b+c+a\)
বা, \(b(1+b)=a+b+c\)
\(\therefore \frac{1}{1+b}=\frac{b}{a+b+c}\) ........... (ii)
এবং \(c^{2}=a+b\), ইহার উভয় পক্ষের সঙ্গে \(c\) যােগ করে পাই,
\(c+c^{2}=a+b+c\)
বা, \(c(1+c)=a+b+c\)
\(\frac{1}{1+c}=\frac{c}{a+b+c}\) ........... (iii)
(i), (ii), (iii) যােগ করে পাই,
\(\frac{1}{1+a}+\frac{1}{1+b}+\frac{1}{1+c}\)
\(=\frac{a}{a+b+c}+\frac{b}{a+b+c}+\frac{c}{a+b+c}\)
\(=\frac{a+b+c}{a+b+c}=1\)
\(\therefore \frac{1}{1+a}+\frac{1}{1+b}+\frac{1}{1+c}=1\) (প্রমাণিত)
(ii) \({x^2}:(by + cz) = {y^2}:(cz + ax) = {z^2}:(ax + by) = 1\)হলে, দেখাই যে, \(\frac{a}{{a + x}} + \frac{b}{{b + y}} + \frac{c}{{c + z}} = 1\)
যেহেতু, \({x^2}:(by + cz) = {y^2}:(cz + ax) = {z^2}:(ax + by) = 1\)
\(\frac{{{x^2}}}{{by + cz}} = \frac{{{y^2}}}{{cz + ax}} = \frac{{{z^2}}}{{ax + by}} = 1\)
\(\therefore {x^2} = by + cz,{y^2} = cz + ax,{z^2} = ax + by\)
এখন, \(\frac{a}{{a + x}} + \frac{b}{{b + y}} + \frac{c}{{c + z}}\)
\(= \frac{{ax}}{{(a + x)x}} + \frac{{by}}{{(b + y)y}} + \frac{{cz}}{{(c + z)z}}\)
\(=\frac{a x}{a x+x^2}+\frac{b y}{b y+y^2}+\frac{c z}{c z+z^2}\)
\( = \frac{{ax}}{{ax + by + cz}} + \frac{{by}}{{by + cz + ax}} + \frac{{cz}}{{cz + ax + by}}\) \(\left[x^2=b y+c z, y^2=c z+a x\right.\)\(\left.z^2=a x+b y\right]\)
\(= \frac{{ax + by + cz}}{{ax + by + cz}}\)
\(= 1\) (প্রমাণিত)

11.

(i) \(\frac{x}{{xa + yb + zc}} = \frac{y}{{ya + zb + xc}} = \frac{z}{{za + xb + yc}}\) এবং \(x + y + z \ne 0\) হলে, দেখাই যে, প্রতিটি অনুপাত \(\frac{1}{{a + b + c}}\)-এর সমান।
\(\frac{x}{x a+y b+z c}=\frac{y}{y a+zb+x c}=\frac{z}{za+x b+y c}\)
\(\because\) প্রতিটি অনুপাত \(=\frac{\text {লম্বগুলির সমষ্টি }} {\text { হরগুলির সমষ্টি }}\)
\(\therefore \quad \frac{x}{x a+y b+zc}=\frac{y}{y a+z b+x c}=\frac{z}{z a+x b+y c}\)
\(=\frac{x+y+z}{x a+y b+zc+y a+z b+x+z a+x b+y c}\) (সংযোজন প্রক্রিয়া)
\(=\frac{x+y+z}{x(a+b+c)+y(a+b+c)+z(a+b+c)}\)
\(=\frac{x+y+z}{(a+b+c)(x+y+z)}\)
\(=\frac{1}{a+b+c} \cdot[\because x+y+z \neq 0]\)
অতএব, প্রতিটি অনুপাতের মান \(\frac{1}{a+b+c}\) (প্রমাণিত)
(ii) \(\frac{{{x^2} - yz}}{a} = \frac{{{y^2} - zx}}{b} = \frac{{{z^2} - xy}}{c}\) হলে, প্রমাণ করি যে, \((a+b+c)(x+y+z)=a x+b y+c z\)
মনে করি, \(\frac{x^{2}-y z}{a}=\frac{y^{2}-z x}{b}=\frac{z^{2}-x y}{c}=\frac{1}{k}(k \neq 0)\) ।[যেখানে k একটি সমানুপাতিক ধ্রুবক]
\(\therefore \quad a=k\left(x^{2}-y z\right), \quad b=k\left(y^{2}-z x\right), \quad c=k\left(z^{2}-x y\right)\)
এখন বামপক্ষ \(=(a+b+c)(x+y+z)\)
\(=\left\{k\left(x^{2}-y z\right)+k\left(y^{2}-z x\right)+k\left(z^{2}-x y\right)\right\}(x+y+z)\)
\(=k\left(x^{2}+y^{2}+z^{2}-x y-y z-z x\right)(x+y+z)\)
\(=k\left(x^{3}+y^{3}+z^{3}-3 x y z\right)\)
এবং ডানপক্ষ \(=a x+b y+c z\)
\(=k\left(x^{2}-y z\right) x+k\left(y^{2}-z x\right) y+k\left(z^{2}-x y\right) z\)
\(=k\left(x^{3}-x y z+y^{3}-x y z+z^{3}-x y z\right)\)
\(=k\left(x^{3}+y^{3}+z^{3}-3 x y z\right)=\) বামপক্ষ।
\(\therefore\) বামপক্ষ = ডানপক্ষ (প্রমাণিত)
(iii) \(\frac{a}{{y + z}} = \frac{b}{{z + x}} = \frac{c}{{x + y}}\) হলে প্রমাণ করি যে, \(\frac{{a(b - c)}}{{{y^2} - {z^2}}} = \frac{{b(c - a)}}{{{z^2} - {x^2}}} = \frac{{c(a - b)}}{{{x^2} - {y^2}}}\)
মনে করা যাক, \(\frac{a}{{y + z}} = \frac{b}{{z + x}} = \frac{c}{{x + y}} = k( \ne 0)\) [যেখানে k একটি সমানুপাতিক ধ্রুবক]
\(\therefore a = k(y + z),b = k(z + x),c = k(x + y)\)
এখন, \(\frac{{a(b - c)}}{{{y^2} - {z^2}}} = \frac{{k(y +z)k\{ (z + x) - (x + y)\} }}{{{y^2} - {z^2}}}\)
\(=\frac{k(y+z) k\{z+x-x-y\}}{y^2-z^2}\)
\(=\frac{-k^2-\left(y^2-z^2\right)}{y^2-z^2}=-k^2\)
আবার, \( = \frac{{{k^2}(z + y)(z - y)}}{{{y^2} - {z^2}}} = \frac{{{k^2}\left( {{z^2} - {y^2}} \right)}}{{{y^2} - {z^2}}} = - {k^2}\)
\(\frac{{b(c - a)}}{{{z^2} - {x^2}}} = \frac{{k(z + x)k\{ (x + y) - (y + z)}\}}{{{z^2} - {x^2}}}\)
\(=\frac{k(z+x) k\{x+y-y-z\}}{z^2-x^2}\)
\( = \frac{{{k^2}(x + z)(x - z)}}{{{z^2} - {x^2}}} = \frac{{{k^2}\left( {{x^2} - {z^2}} \right)}}{{{z^2} - {x^2}}} = - {k^2}\)
\(=\frac{-k^2\left(z^2-x^2\right)}{\left(z^2-x^2\right)}=-k^2\)
আবার, \(\frac{{c(a - b)}}{{{x^2} - {y^2}}} = \frac{{k(x + y)k\{ (y + z) - (z + x)\} }}{{{x^2} - {y^2}}} = \frac{{{k^2}(y + x)(y - x)}}{{{x^2} - {y^2}}} = \frac{{{k^2}\left( {{y^2} - {x^2}} \right)}}{{{x^2} - {y^2}}} = - {k^2}\)
\(=\frac{-k^2\left(x^2-y^2\right)}{\left(x^2-y^2\right)}=-k^2\)
সুতরাং প্রমাণিত হল যে, \(\frac{{a(b - c)}}{{{y^2} - {z^2}}} = \frac{{b(c - a)}}{{{z^2} - {x^2}}} = \frac{{c(a - b)}}{{{x^2} - {y^2}}}\)

12. অতিসংক্ষিপ্ত প্রশ্ন উত্তরধর্মী (V.S.A)

(A) বহুবিকল্পীয় প্রশ্ন (M.C.Q)

(i) 3, 4-এর 6-এর চতুর্থ সমানুপাতী - (a) 8 (b) 10 (c) 12 (d) 24
চতুর্থ সমানুপাতী \( =\frac{ \text {দ্বিতীয় পদ} \times \text {তৃতীয় পদ}}{ \text {প্রথম পদ}}\)
\(\frac{{4 \times 6}}{3} = 8\)
\(\therefore\) (a) উত্তরটি ঠিক
(ii) 8 ও 12-এর তৃতীয় সমানুপাতী - (a) 12 (b) 16 (c) 18 (d) 20
তৃতীয় সমানুপাতী = \(\frac{{12 \times 12}}{8}\) = 18
(c) উত্তরটি ঠিক।
Koshe Dekhi 5.3 Class 10 | অনুপাত সমানুপাত কষে দেখি ৫.৩ | Class 10 Maths Solution WBBSE
Madhyamik Previous Year Question Paper | মাধ্যমিক বিগত বছরের অঙ্ক প্রশ্নের উত্তরপত্র
আজই Install করুন Chatra Mitra
(iii) 16 এবং 25-এর মধ্যসমানুপাতী - (a) 400 (b) 100 (c) 20 (d) 40
মধ্য সমানুপাতী = \(\sqrt{16 \times 25} = 4 \times 5=20\)
(iv) a একটি ধনাত্মক সংখ্যা এবং \(a:\frac{{27}}{{64}} = \frac{3}{4}:a\) হলে a-এর মান - \({\rm{ (a) }}\frac{{81}}{{256}}{\rm{ (b) }}9{\rm{ (c) }}\frac{9}{{16}}{\rm{ (d) }}\frac{{16}}{9}\)
যেহেতু,\(a: \frac{27}{64}=\frac{3}{4}: a\),
সুতরাং, \(\frac{a}{\frac{27}{64}}=\frac{\frac{3}{4}}{a}\)
বা, \(a^{2}=\frac{3}{4} \times \frac{27}{64}=\frac{81}{256}\)
বা, \(a^{2}=\left(\pm \frac{9}{16}\right)^{2}\)
\(\therefore a=\pm \frac{9}{16}\)
যেহেতু, \(a\) একটি ধনাত্মক সংখ্যা, সুতরাং \(a\)-এর ধনাত্মক মানটি গ্রহণযােগ্য এবং ঋণাত্মক মানটি বর্জনীয়।
\(a\)-এর নির্ণেয় মান \(=\frac{+9}{16}\) অর্থাৎ, \(a\)-এর নির্ণেয় মান \(=\frac{9}{16}\)
উত্তর : \(a\)-এর নির্ণেয় মান \(=\frac{9}{16}\) (c)
(v) \(2a = 3b = 4c\) হলে \(a : b : c\) হবে - (a) 3 : 4 : 6 (b) 4 : 3 : 6 (c) 3 : 6 : 4 (d) 6 : 4 : 3
মনেকরি, \(2a = 3b = 4c = k\) \((k \neq 0)\)[যেখানে k একটি সমানুপাতিক ধ্রুবক]
\(\therefore a = \frac{k}{2},b = \frac{k}{3},c = \frac{k}{4}\)
\(\therefore\) a : b : c
\(=\frac{k}{2}: \frac{k}{3}: \frac{k}{4}\)
\( = \frac{{k \times 6}}{{2 \times 6}}:\frac{{k \times 4}}{{3 \times 4}}:\frac{{k \times 3}}{{4 \times 3}}\)
\(=\frac{6 k}{12}: \frac{4 k}{12}: \frac{3 k}{12}\)
\(= 6k : 4k : 3k\)
\(= 6 : 4 : 3 \)
\(\therefore\) (d) উত্তরটি ঠিক।

(B) নীচের বিবৃতিগুলি সত্য না মিথ্যা লিখি :

(i) \(ab : c^2, bc : a^2\) এবং\( ca : b^2\)-এর যৌগিক অনুপাত 1 : 1
সত্য।
কারণ, \(a b \times b c \times c a: c^2 \times a^2 \times b^2\)
\(\Rightarrow a^2 \times b^2 \times c^2: a^2 \times b^2 \times c^2\)
\(=1: 1\)
(ii) \({x^3}y,{x^2}{y^2}\) এবং \({xy}^3\) ক্ৰমিক সমানুপাতী।
সত্য।
কারণ, \(\frac{x^3 y^2}{x^2 y^2}=\frac{x^2 y^2}{x y^3} \Rightarrow \frac{x}{y}=\frac{x}{y}\)

(C) শূন্যস্থান পূরণ করো :

(i) তিনটি ক্রমিক সমানুপাতী ধনাত্মক সংখ্যার গুনফল 64 হলে, তাদের মধ্যসমানুপাতী = __________
মনে করি, তিনটি ক্রমিক সমানুপাতী সংখ্যা যথাক্রমে \(x, y, z\)
\(\therefore \frac{x}{y}=\frac{y}{z}\)
\(\therefore y^{2}=x z \ldots\) (i)
শর্তানুসারে, \(x y z=64\)
বা, \((x z) \cdot y=64\)
বা, \(y^{2} \cdot y=64\) [(i) থেকে \(x z=y^{2}\) বসিয়ে]
বা, \(y^{3}=64=4^{3}\)
\(\therefore y=4\)
\(\therefore\) মধ্যসমানুপাতী \(= 4\).
\(\therefore\) মধ্যসমানুপাতী সংখ্যার মান \(= 4\).
(ii) \(a : 2 = b : 5 = c : 8\) হলে a-এর \(50\% = b\)-এর \( 20\% = c\)-এর _____%.
12.5% ।
\(\frac{a}{2}=\frac{b}{5}=\frac{c}{8}\)
\(\Rightarrow \frac{a}{2} \times 100 \%=\frac{b}{5} \times 100 \%=\frac{c}{8} \times 100 \%\)
\(\Rightarrow a \times 50 \%=b \times 20 \%=c \times 12.5 \%\)
(iii) \((x+2)\) এবং \((x-3)\)-এর মধ্য সমানুপাতী \(x\) হলে, \(x\)-এর মান ______
\(\Rightarrow-6\)
\(\sqrt{(x+2)(x-3)}=x\)
\(\Rightarrow x^2-3 x+2 x-6=x^2\)
\(\Rightarrow-x=6\)
\(\Rightarrow x=-6\)

13. সংক্ষিপ্ত উত্তরধৰ্মী প্ৰশ্ন (S.A.)

(i) \(\frac{a}{2} = \frac{b}{3} = \frac{c}{4} = \frac{{2a - 3b + 4c}}{p}\) হলে p এর মান নির্ণয় করি।
\(\frac{a}{2} = \frac{b}{3} = \frac{c}{4} = \frac{{2a - 3b + 4c}}{p} = k\) (ধরি)\((k \neq 0)\)[যেখানে k একটি সমানুপাতিক ধ্রুবক]
\(\therefore a = 2k,b = 3k,c = 4k\)
\(\therefore 2.2 \mathrm{k}-3.3 \mathrm{k}+4.4 \mathrm{k}=\mathrm{pk} \Rightarrow 11 \mathrm{k}=\mathrm{pk} \Rightarrow \mathrm{p} = 11\)
\(\therefore\) p-এর মান \(= 11\)
(ii) \(\frac{{3x - 5y}}{{3x + 5y}} = \frac{1}{2}\) হলে \(\frac{{3{x^2} - 5{y^2}}}{{3{x^2} + 5{y^2}}}\) এর মান নির্ণয় করি ।
\(\frac{{3x - 5y}}{{3x + 5y}} = \frac{1}{2}\)
বা, \(6 x-10 y=3 x+5 y\)
বা, \(6 x-3 y=5 y+10 y\)
বা, 3 \(x=15 y\)
বা, \(x=5 y\)
\(\therefore\) \(\frac{{3{x^2} - 5{y^2}}}{{3{x^2} + 5{y^2}}}\)
\(= \frac{{3{{(5y)}^2} - 5{y^2}}}{{3{{(5y)}^2} + 5{y^2}}}\)
\(= \frac{{75{y^2} - 5{y^2}}}{{75{y^2} + 5{y^2}}}\)
\(= \frac{{70}}{{80}}\)
\(= \frac{7}{8}\)
\(\therefore\) নির্ণেয় মান \(= \frac{7}{8}\)
(iii) \(a : b = 3 : 4\) এবং \(x: y = 5 : 7\) হলে \((3ax - by) : (4by – 7ax)\) কত নির্ণয় করি?
ধরি \(a = 3k, b = 4k\) এবং x = 5p ও y = 7p
\(\therefore\) (3ax - by) : (4by - 7ax)
\(=(3\times 3k \times 5 p-4 k \times 7 p):(4 \times 4 k \times 7 p-7 \times 3k \times 5 p)\)
\(=(45kp-28kp):(112kp-105kp)\)\(=k P(45-28): k p(112-105)\)
\(=(45-28):(112-105)\)
\(=17: 7\)
\(\therefore\) নির্ণেয় মান \(=17: 7\)
(iv) \(x , 12, y, 27\) ক্রমিক সমানুপাতী হলে \(x\) ও \(y\)-এর ধনাত্মক মান নির্ণয় করি।
\(\frac{x}{{12}} = \frac{{12}}{y} = \frac{y}{{27}}\)
\(\frac{{12}}{y} = \frac{y}{{27}}\)
বা, \({y^2} = 27 \times 12\)
বা, \(y = \sqrt {12 \times 27} = \sqrt {324} = 18\)
বা, \(\frac{x}{{12}} = \frac{{12}}{y}\)
বা, \(xy = 12 \times 12\)
বা, \(x = \frac{{144}}{{18}} = 8\) \([\because y=18]\)
\(\therefore\) \(x\)-এর মান 8 ও y-এর মান 18
(v) \(a : b = 3 : 2\) এবং \(b : c = 3 : 2\) হলে \(a + b : b + c\) কত নির্ণয় করি।
\(a : b = 3 : 2 = 9 : 6\)
\(b : c = 3 : 2 = 6 : 4\)
\(\therefore a: b: c=9: 6: 4\)
ধরি \(a = 9k, b = 6k, c = 4k\)\((k \neq 0)\)[যেখানে k একটি সমানুপাতিক ধ্রুবক]
\(a + b : b + c = 9k + 6k : 6x + 4k\)
\(= 15k : 10k = 3 : 2\)
Koshe Dekhi 5.3 Class 10 | অনুপাত সমানুপাত কষে দেখি ৫.৩ | Class 10 Maths Solution WBBSE
Madhyamik Previous Year Question Paper | মাধ্যমিক বিগত বছরের অঙ্ক প্রশ্নের উত্তরপত্র
আজই Install করুন Chatra Mitra
এই Page টি বা এই Website টির কোন প্রকার বিষয়বস্তু কপি করা বা সম্পাদনা করা নিষিদ্ধ। ভারতীয় Copywright আইন 1957 এর ধারা 63 অনুযায়ী, এই ফাইলটির সমস্ত অধিকার 'ছাত্র মিত্র Mathematics' অ্যাপ দ্বারা সংরক্ষিত। ছাত্র মিত্রের অনুমতি ছাড়া, এই Page টি বা এই Website টির কোন প্রকার বিষয়বস্তু কপি করা বা সম্পাদনা করা আইনত দন্ডনীয় অপরাধ। কেউ ছাত্র মিত্রের অনুমতি ছাড়া, এই Page টি বা এই Website টির কোন প্রকার বিষয়বস্তু কপি বা সম্পাদনা করলে ছাত্র মিত্র কতৃপক্ষ তার বিরুদ্ধে সকল প্রকার কঠোর আইনি পদক্ষেপ করবে।

Leave a Comment

Your email address will not be published. Required fields are marked *

Share this page using:

Exit mobile version